summaryrefslogtreecommitdiff
path: root/macros/latex/contrib/xsim/doc/examples/xsim.texsx-369803.tex
blob: 58b6800f93728a00e85e18252cff6563d00e63b6 (plain)
1
2
3
4
5
6
7
8
9
10
11
12
13
14
15
16
17
18
19
20
21
22
23
24
25
26
27
28
29
30
31
32
33
34
35
36
37
38
39
40
41
42
43
44
45
46
47
48
49
50
% https://tex.stackexchange.com/q/369803
\documentclass[a4paper,parskip=half]{scrartcl}
\usepackage[utf8]{inputenc}
\usepackage[ngerman]{babel}

\usepackage{amsmath}
\usepackage{amsthm} 
\usepackage{amsfonts}
\usepackage{amssymb}

\usepackage[left=2cm,right=2.5cm,top=2.5cm,bottom=2cm]{geometry}

\usepackage{xsim,siunitx}
\DeclareExerciseTagging{difficulty}
\DeclareExerciseEnvironmentTemplate{custom}
  {%
    \subsection*
      {%
        \XSIMmixedcase{\GetExerciseName}\nobreakspace
        \GetExerciseProperty{counter}%
        \IfInsideSolutionF
          {%
            \IfExercisePropertySetT{subtitle}
              { {\normalfont(\GetExerciseProperty{subtitle})}}%
          }%
      }%
  }{}

\xsimsetup{
  exercise/name = \XSIMtranslate{question} ,
  exercise/template=custom ,
  solution/print=true
}

\begin{document}

\begin{exercise}[ID=wdsw, subtitle = Widerstandswürfel , difficulty = 2]
  Gegeben ist ein Würfel, wobei jede der Kanten einen Widerstand von $R =
  \SI{1}{\ohm}$ hat.
  
  Wie groß ist der Widerstand entlang einer Raumdiagonale?
\end{exercise}
\begin{solution}
  Wir wollen den Widerstand zwischen den Punkten $X$ und $Y$ bestimmen, also
  entlang der Raumdiagonale (siehe Abb. \ref{fig:wdsws1}). Weil die
  Raumdiagonale eine Symmetrieachse ist, sollte das Problem symmetrisch sein,
  und deswegen eine recht einfache Lösung haben.
\end{solution}

\end{document}